Difference between revisions of "2004 AMC 12A Problems/Problem 5"

(Undo revision 50681 by Heliootrope (talk))
(See also)
Line 13: Line 13:
 
==See also==
 
==See also==
 
{{AMC12 box|year=2004|ab=A|num-b=4|num-a=6}}
 
{{AMC12 box|year=2004|ab=A|num-b=4|num-a=6}}
 +
{{MAA Notice}}

Revision as of 20:15, 3 July 2013

Problem

The graph of the line $y=mx+b$ is shown. Which of the following is true?

2004 AMC 12A Problem 5.png

$\mathrm {(A)} mb<-1 \qquad \mathrm {(B)} -1<mb<0 \qquad \mathrm {(C)} mb=0 \qquad \mathrm {(D)}$ $0<mb<1 \qquad \mathrm {(E)} mb>1$

Solution

It looks like it has a slope of $-\dfrac{1}{2}$ and is shifted $\dfrac{4}{5}$ up.

$\dfrac{4}{5}\cdot \dfrac{-1}{2}=\dfrac{-4}{10} \Rightarrow \mathrm {(B)}$

See also

2004 AMC 12A (ProblemsAnswer KeyResources)
Preceded by
Problem 4
Followed by
Problem 6
1 2 3 4 5 6 7 8 9 10 11 12 13 14 15 16 17 18 19 20 21 22 23 24 25
All AMC 12 Problems and Solutions

The problems on this page are copyrighted by the Mathematical Association of America's American Mathematics Competitions. AMC logo.png